Difference between revisions of "2019 AMC 12B Problems/Problem 22"

(Created page with "==Problem== ==Solution== ==See Also== {{AMC12 box|year=2019|ab=B|num-b=21|num-a=23}}")
 
(Problem: Added problem statement.)
Line 1: Line 1:
 
==Problem==
 
==Problem==
 +
Define a sequence recursively by <math>x_0 = 5</math> and
 +
 +
<math>x_{n+1} = \frac{x_n^2 + 5x_n + 4}{x_n + 6}</math>
 +
 +
for all nonnegative integers <math>n</math>. Let <math>m</math> be the least positive integer such that <math>x_m \leq 4 + \frac{1}{2^{20}}</math>.
 +
 +
In which of the following intervals does <math>m</math> lie?
  
 
==Solution==
 
==Solution==

Revision as of 17:38, 14 February 2019

Problem

Define a sequence recursively by $x_0 = 5$ and

$x_{n+1} = \frac{x_n^2 + 5x_n + 4}{x_n + 6}$

for all nonnegative integers $n$. Let $m$ be the least positive integer such that $x_m \leq 4 + \frac{1}{2^{20}}$.

In which of the following intervals does $m$ lie?

Solution

See Also

2019 AMC 12B (ProblemsAnswer KeyResources)
Preceded by
Problem 21
Followed by
Problem 23
1 2 3 4 5 6 7 8 9 10 11 12 13 14 15 16 17 18 19 20 21 22 23 24 25
All AMC 12 Problems and Solutions